Which corectly renames 7/8 and 5/6 using a common denominator

Answers

Answer 1

The 7/8 and 5/6, when renamed using a common denominator of 24, become 21/24 and 20/24

How we find the common denominator?

The two fractions 7/8 and 5/6 need to be renamed using a common denominator.

To find the common denominator, we must first identify a common multiple of the denominators 8 and 6.

The smallest common multiple of 8 and 6 is 24. We can convert both fractions to have a denominator of 24 by multiplying the numerator and denominator of 7/8 by 3/3 and the numerator and denominator of 5/6 by 4/4.

This results in 21/24 and 20/24, respectively.  

Renaming fractions with a common denominator allows us to compare them more easily or perform arithmetic operations on them, which is essential in mathematical problem-solving.

Learn more about Common denominator

brainly.com/question/29775115

#SPJ11


Related Questions

A floor plan of a house was drawn using a scale of 1 inch:5 feet. if the kitchen is drawn 2 1/2 inches by 3 inches, what are the dimensions of the actual kitchen?

Answers

If the kitchen is drawn 2 1/2 inches by 3 inches, the actual dimensions of the kitchen are 12.5 feet by 15 feet.

The given scale of 1 inch:5 feet means that for every 1 inch on the floor plan, the actual length in real life is 5 feet. To find the actual dimensions of the kitchen, we need to convert the length and width of the kitchen on the floor plan into real-life measurements.

The length of the kitchen on the floor plan is 2 1/2 inches, which in real life would be:

2.5 inches x 5 feet/1 inch = 12.5 feet

Similarly, the width of the kitchen on the floor plan is 3 inches, which in real life would be:

3 inches x 5 feet/1 inch = 15 feet

To verify this result, we can also use the scale to convert the actual dimensions of the kitchen back into the measurements on the floor plan. The length of the kitchen in real life is 12.5 feet, which on the floor plan would be:

12.5 feet x 1 inch/5 feet = 2.5 inches

Similarly, the width of the kitchen in real life is 15 feet, which on the floor plan would be:

15 feet x 1 inch/5 feet = 3 inches

As expected, these measurements match the dimensions of the kitchen as drawn on the floor plan.

To learn more about scale click on,

https://brainly.com/question/23864395

#SPJ4

Can someone help me asap? It’s due today!! Show work! I will give brainliest if it’s correct and has work

Make a probability table!

Answers

The probability of choosing randomly with replacement an H or P in either selection is derived to be equal to 0.16 which makes the last option correct.

What is probability

The probability of an event occurring is the fraction of the number of required outcome divided by the total number of possible outcomes.

The total possible outcome = 5

the event of selecting H = 1

probability of selecting H= 1/5

the event of selecting P = 2

probability of selecting H= 2/5

probability of choosing an H or P in either selection = 1/5 × 2/5 + 2/5 × 1/5

probability of choosing an H or P in either selection = 4/25

probability of choosing an H or P in either selection = 0.16

Therefore, the probability of choosing randomly with replacement an H or P in either selection is derived to be equal to 0.16

Read more about probability here:https://brainly.com/question/251701

#SPJ1

Suppose C is any curve from (0,0,0) to (1,1,1) and F (x, y, z) = (1z + 5y) i + (1z + 5x)j + (1y + 1x)k. After confirming that F is conservative, compute a potential function f for F with constant term 0.

Answers

The potential function f(x, y, z) evaluated at the endpoints of C gives the same result:

f(1, 1, 1) - f(0, 0, 0) = (1 + 5 + 5 + 1/2 + 1/2) - 0 = 12

This confirms that f is indeed the potential function for F.

How to confirm that F is conservative?

To confirm that F is conservative, we need to check if its curl is zero. The curl of F is given by:

[tex]curl(F) = (∂F_z/∂y - ∂F_y/∂z) i + (∂F_x/∂z - ∂F_z/∂x) j + (∂F_y/∂x - ∂F_x/∂y) k[/tex]

Substituting F(x, y, z) = (1z + 5y) i + (1z + 5x)j + (1y + 1x)k into the above equation, we get:

curl(F) = 0i + 0j + 0k

The potential function f for F, we need to integrate F along any path from (0,0,0) to (1,1,1). Let C be the path given by the line segment connecting (0,0,0) and (1,1,1).

The parametric equations of C are:

x = ty = tz = t

where 0 ≤ t ≤ 1.

We need to evaluate the line integral ∫CF.dr, where r(t) = ti + tj + tk is the position vector of C at time t. The potential function f is defined as the line integral of F from (0,0,0) to (x,y,z), so we need to find an antiderivative of F to evaluate this integral.

The antiderivative of F is:

[tex]f(x, y, z) = z + 5xy + 5xz + (1/2)y^2 + (1/2)x^2 + C[/tex]

where C is a constant of integration. We want f to have a constant term of 0, so we choose C = 0.

[tex]f(x, y, z) = z + 5xy + 5xz + (1/2)y^2 + (1/2)x^2[/tex]

Now we can evaluate the line integral ∫CF.dr by substituting the parametric equations of C into F and taking the dot product with the differential of r(t):

[tex]F(r(t)).dr/dt = ((t+5t) i + (t+5t)j + (t+t)k) . (i+j+k) dt = (7t) dt[/tex]

Integrating from t=0 to t=1, we get:

[tex]∫CF.dr = ∫0^1 7t dt = 7/2[/tex]

Learn more about F is conservative

brainly.com/question/24314070

#SPJ11

Lisa has 9 rings in her jewelry box. Five are gold and 4 are silver. If she randomly selects 3 rings to wear to a party, find each probability. P(2 silver or 2 gold)

Answers

The probability of selecting 2 silver rings or 2 gold rings is 3/28.

How to find the probability of selecting 2 silver rings or 2 gold rings?

To find the probability of selecting 2 silver rings or 2 gold rings, we need to find the probability of each event separately and then add them.

Probability of selecting 2 silver rings:

There are 4 silver rings out of 9 total, so the probability of selecting a silver ring on the first draw is 4/9. After the first ring is selected, there are 3 silver rings left out of 8 total, so the probability of selecting a second silver ring is 3/8. Finally, after two silver rings have been selected, there are 2 silver rings left out of 7 total, so the probability of selecting a third silver ring is 2/7. Therefore, the probability of selecting 2 silver rings is:

(4/9) * (3/8) * (2/7) = 24/504 = 1/21

Probability of selecting 2 gold rings:

Similarly, there are 5 gold rings out of 9 total, so the probability of selecting a gold ring on the first draw is 5/9. After the first ring is selected, there are 4 gold rings left out of 8 total, so the probability of selecting a second gold ring is 4/8 = 1/2. Finally, after two gold rings have been selected, there are 3 gold rings left out of 7 total, so the probability of selecting a third gold ring is 3/7. Therefore, the probability of selecting 2 gold rings is:

(5/9) * (1/2) * (3/7) = 15/126 = 5/42

Adding the probabilities of selecting 2 silver rings or 2 gold rings, we get:

P(2 silver or 2 gold) = P(2 silver) + P(2 gold) = 1/21 + 5/42 = 3/28

Therefore, the probability of selecting 2 silver rings or 2 gold rings is 3/28.

Learn more about probability

brainly.com/question/30034780

#SPJ11

Mike receives a bonus every year. His bonus is calculated as 3 percent of his company's total profits. If he estimates his company's total profits to be between $500,000 and $650,000, which inequality best represents Mike's bonus, B, for the year?

Answers

Mike's bonus for the year is between $15,000 and $19,500.

The inequality that best represents Mike's bonus, B, for the year is:

$15,000 [tex]\leq B \leq[/tex] 19,500$

to see why, we are able to use the given data that Mike's bonus is calculated as 3 percent of his corporation's overall profits.

If we let P be the organization's general income, then Mike's bonus B can be expressed as:

$B = 0.03P$

We recognise that the organization's total profits are between $500,000 and $650,000, so we will write:

$500,000 [tex]\leq P \leq[/tex] 650,000$

Substituting this inequality into the equation for Mike's bonus, we get:

$15,000 [tex]\leq B \leq[/tex] 19,500$

Therefore, Mike's bonus for the year is between $15,000 and $19,500.

Learn more about inequality:-

https://brainly.com/question/30238989

#SPJ4




CAN SOMEONE PLEASE HELP ME ILL GIVE BRAINLIST












Mai and Elena are shopping


for back-to-school clothes. They found a skirt that originally cost $30


on a 15% off sale rack. Today, the store is offering an additional 15% off. To find the new price of


the skirt, in dollars, Mai says they need to calculate 30. 0. 85 0. 85. Elena says they can just


multiply 30. 0. 70.


1. How much will the skirt cost using Mai's method?


2. How much will the skirt cost using Elena's method?


3. Explain why the expressions used by Mai and Elena give different prices for the skirt. Which


method is correct?

Answers

1. The skirt cost using Mai's method is $21.68.

2. The skirt cost using Elena's method is $21.

3. Mai's method is correct because she correctly calculates the discounts sequentially while Elena combines the discount.

We'll examine the methods suggested by Mai and Elena for finding the new price of the skirt and determine which one is correct.

1. Using Mai's method (30 x 0.85 x 0.85):

1: Calculate the first 15% off discount: 30 x 0.85 = 25.50

2: Calculate the additional 15% off discount: 25.50 x 0.85 = 21.675

So, the skirt will cost $21.68 using Mai's method (rounded to the nearest cent).

2. Using Elena's method (30 x 0.70):

Elena suggests taking 30% off the original price. To do this, we multiply the original price by 0.70:

30 x 0.70 = 21

So, the skirt will cost $21 using Elena's method.

3. Explanation of the difference in expressions and the correct method:

Mai's method is correct because she correctly calculates the discounts sequentially. The first 15% off is applied to the original price, and then the additional 15% off is applied to the reduced price. This results in a final price of $21.68.

Elena's method is incorrect because she combines the two discounts into a single 30% off, which does not accurately reflect the sequential discounts. By doing this, she finds a final price of $21, which is not correct.

In conclusion, Mai's method (30 x 0.85 x 0.85) is the correct way to calculate the new price of the skirt, resulting in a final cost of $21.68.

Learn more about Cost:

https://brainly.com/question/19104371

#SPJ11

Solve for x trigonometry

Answers

Step-by-step explanation:

We are given an angle opposite of the side length x and the hypotenuse 10.

Use SOHCAHTOA, use Sin

[tex] \sin( \alpha ) = \frac{o}{h} [/tex]

We the angle is 20

and the hypotenuse is 10 and the opposite is x.

[tex] \sin(20) = \frac{x}{10} [/tex]

[tex]10 \sin(20) = x[/tex]

And we get

[tex]x = 3.42[/tex]

Which answer gives the correct transformation of P(x) to get to I(x)?



A. ) I(x)=P(1/2x)


B. ) I(x)=P(2x)


C. ) I(x)=1/2P(x)


D. ) I(x)=2P(x)

Answers

The answer that gives the correct transformation of P(x) to get to I(x) is option D) I(x) = 2P(x).

This means that the function I(x) is obtained by multiplying the function P(x) by 2.

To understand why this is the correct transformation, let's consider an example:

Suppose P(x) represents the number of items produced by a factory in x hours. If we want to find the number of items produced by the factory in 2x hours, we can use the transformation I(x) = 2P(x). This is because the rate of production is constant, so in twice the time, the factory will produce twice the number of items. Therefore, multiplying the function P(x) by 2 gives us the function I(x) that represents the number of items produced by the factory in 2x hours.

Option A) I(x) = P(1/2x) means that we are compressing the function P(x) horizontally, which would result in a faster rate of change. This transformation does not make sense in the context of the problem and is not the correct transformation.

Option B) I(x) = P(2x) means that we are stretching the function P(x) horizontally, which would result in a slower rate of change. This transformation also does not make sense in the context of the problem and is not the correct transformation.

Option C) I(x) = 1/2P(x) means that we are reducing the function P(x) by half, which would result in a slower rate of change. This transformation does not match the problem statement and is not the correct transformation.

To learn more about  transformation visit:

https://brainly.com/question/29641135

#SPJ11

Drag each tile to its equivalent measure, rounded to the nearest tenth.



19. 810. 222. 715. 4



Measure Equivalent



4 in.



cm



7 kg



lb



6 gal



L



65 ft



m

Answers

The given value of 19 is not a unit of measurement, so it cannot be converted to an equivalent measure.

How to drag each tile to its equivalent measure, rounded to the nearest tenth?

4 in. - 10.2 cm

7 kg - 15.4 lb

6 gal - 22.7 L

5 ft - 1.5 m

Note: The given value of 19 is not a unit of measurement, so it cannot be converted to an equivalent measure.

Learn more about measurement

brainly.com/question/4725561

#SPJ11

Sarah has a solid wooden cube with a length of 4/5 cm. From each of its 8 corners, she cuts out a smaller cube with a length of 1/5 cm. What is the volume of the block after cutting out the smaller cubes?

Answers

The volume of the block after cutting out the smaller cubes is 56/125 cubic centimeters.

The initial volume of the solid wooden cube is given by:

V_initial = (4/5 cm)³ = 64/125 cm³

To find the volume of each of the 8 smaller cubes cut out from the corners, we can use the formula:

V_small cube = (1/5 cm)³= 1/125 cm³

Since we cut out 8 smaller cubes, the total volume of the smaller cubes is:

V_small cubes = 8 x (1/125 cm³) = 8/125 cm³

To find the final volume of the block after cutting out the smaller cubes, we can subtract the volume of the smaller cubes from the initial volume of the block:

V_final = V_initial - V_small cubes

Substituting the values we obtained earlier, we get:

V_final = (64/125 cm³) - (8/125 cm³) = 56/125 cm³

Know more about volume here:

https://brainly.com/question/1578538

#SPJ11

Some students were asked how many pens they were carrying in their backpacks. The data is given in this frequency table. What is the mean number of pens carried by these students in their backpacks?

A. 2

B. 3. 5

C. 4

D. 5. 5

Answers

The mean number of pens carried by these students in their backpacks is:

122 / 30 = 4.07 (rounded to two decimal places)

So the answer is closest to option C, which is 4.

What is the mean number of pens carried by students in their backpacks given the following frequency table?

To find the mean number of pens carried by the students, we need to calculate the sum of all the pens and divide by the total number of students. We can use the frequency table to calculate the sum of all the pens as follows:

2 x 3 + 3 x 6 + 4 x 10 + 5 x 8 + 6 x 3 = 6 + 18 + 40 + 40 + 18 = 122

The total number of students is the sum of the frequencies, which is:

3 + 6 + 10 + 8 + 3 = 30

The mean number of pens carried by these students in their backpacks is:

122 / 30 = 4.07 (rounded to two decimal places)

Learn more about Frequency table

brainly.com/question/28931302

#SPJ11

Find the area of the regular polygon. Round your answer to the nearest whole number of square units.

The area is about square units. ​

Answers

The area of the regular pentagon is about 9 square units.

To find the area of a regular polygon, we need to know the length of the apothem and the perimeter of the polygon. The apothem is the distance from the center of the polygon to the midpoint of one of its sides, and the perimeter is the sum of the lengths of all the sides.

Since the polygon is regular, all of its sides have the same length. Let's call that length "s". We also know that the polygon has 5 sides, so it is a pentagon. To find the perimeter, we can simply multiply the length of one side by the number of sides:

Perimeter = 5s

Now, to find the apothem, we can use the formula:

Apothem = (s/2) x tan(180/n)

Where "n" is the number of sides. For our pentagon, n = 5, so we have:

Apothem = (s/2) x tan(36)

We can simplify this a bit by noting that tan(36) is equal to approximately 0.7265. So we have:

Apothem = (s/2) x 0.7265

Now we have everything we need to find the area. The formula for the area of a regular polygon is:

Area = (1/2) x Perimeter x Apothem

Substituting in the values we found earlier, we have:

Area = (1/2) x 5s x (s/2) x 0.7265

Simplifying this expression, we get:

Area = (s^2 x 1.8176)

Rounding to the nearest whole number of square units, we have:

Area = 9

So the area of the regular pentagon is about 9 square units.

To know more about pentagon, refer here:

https://brainly.com/question/27476#

#SPJ11

What is the vertex and x-intercepts of -6x^2-50x+3085. 25

Answers

The vertex and x-intercepts of -6x^2-50x+3085. 25 are approximately -42.60 and 30.97.

To find the vertex and x-intercepts of the quadratic function -6x^2-50x+3085.25, we first need to express it in standard form -6x^2-50x+3085.25 = -6(x^2+8.33x-514.21)

So the x-intercepts are approximately -42.60 and 30.97.

We can complete the square to find the vertex of the parabola:

-6(x^2+8.33x-514.21) = -6[(x+4.165)^2-575.641]

-6(x^2+8.33x-514.21) = -6(x+4.165)^2+3453.844

So the vertex is at (-4.165, 575.844).

To find the x-intercepts, we can set y = 0 and solve for x:

-6x^2-50x+3085.25 = 0

Dividing both sides by -2.25 to simplify, we get:

2.6667x^2+22.2222x-1372.2222 = 0

Using the quadratic formula, we get:

x = (-22.2222 ± sqrt(22.2222^2-4(2.6667)(-1372.2222))) / (2(2.6667))

x = (-22.2222 ± sqrt(37511.1116)) / 5.3334

x = (-22.2222 ± 193.7262) / 5.3334

So the x-intercepts are approximately -42.60 and 30.97.

To learn more about “vertex” refer to the https://brainly.com/question/29476657

#SPJ11

In an isosceles triangle, the measure of a base angle is 65. Find the number of degrees in the measure of the vertex angle

Answers

The number of degrees in the measure of the vertex angle is 50 degrees.

An isosceles triangle has two equal sides and two equal base angles. In your question, the measure of a base angle is 65 degrees. To find the measure of the vertex angle, we'll use the fact that the sum of angles in any triangle is always 180 degrees.

Since both base angles are equal, their combined measure is 2 * 65 = 130 degrees. Now, we subtract the sum of the base angles from the total angle measure of the triangle:

180 degrees (total angle measure) - 130 degrees (sum of base angles) = 50 degrees.

So, the measure of the vertex angle in the isosceles triangle is 50 degrees. In summary, when given the measure of a base angle in an isosceles triangle, we can use the triangle's angle sum property to find the measure of the vertex angle.

Learn more about isosceles triangle here: https://brainly.com/question/29793403

#SPJ11

The height, h, in feet of a ball suspended from a spring as a function of time, t, in seconds can be modeled by the equation h = negative 2 sine (pi (t one-half)) 5. which of the following equations can also model this situation? h = negative 2 cosine (pi t) 5 h = negative 2 cosine (pi (t one-half)) 5 h = 2 cosine (pi t) 5 h = 2 cosine (pi (t one-half)) 5

Answers

The correct answer for the equation is  [tex]h = -2cos(\pi t) + 5[/tex] . The correct option is (1)

Given:

[tex]h= -2sin(\pi\tfrac{t}{2} )[/tex]

Examine the answer choices:

[tex]h = -2cos(\pi t) + 5[/tex]

Amplitude: |-2| = 2 (same as the given equation)

Frequency: π (same as the given equation)

Phase Shift: None (different from the given equation)

[tex]h = -2cos(\pi (t/2)) + 5[/tex]

Amplitude: |-2| = 2 (same as the given equation)

Frequency: π/2 (different from the given equation)

Phase Shift: None (different from the given equation)

[tex]h = 2cos(\pi t) + 5[/tex]

Amplitude: |2| = 2 (different from the given equation)

Frequency: π (same as the given equation)

Phase Shift: None (different from the given equation)

[tex]h = 2cos(\pi(t/2)) + 5[/tex]

Amplitude: |2| = 2 (different from the given equation)

Frequency: π/2 (different from the given equation)

Phase Shift: None (different from the given equation)

The correct equation is [tex]h = -2cos(\pi t) + 5[/tex]  .The correct option is (1).

Learn more about Equation here:

https://brainly.com/question/27767092

#SPJ12

Let AX = B be a consistent linear system with 12 equations and 8 variables. If the solution of the system contains 3 free variables, then what is the rank of the coefficient matrix A?

Answers

The rank of the coefficient matrix A is 5.

How to determined the matrix?

Since the system AX = B is consistent and has 12 equations and 8 variables, the rank of the coefficient matrix A must be less than or equal to 8 (the number of variables).

If the solution of the system contains 3 free variables, it means that the dimension of the null-space of A is 3. By the rank-nullity theorem,

we know that the dimension of the null-space of A plus the rank of A is equal to the number of columns of A (which is 8 in this case).

Therefore, we have:

rank(A) + dim(null(A)) = 8

rank(A) + 3 = 8

rank(A) = 5

So, the rank of the coefficient matrix A is 5.

Learn more about matrix

brainly.com/question/9967572

#SPJ11

What expression represents the volume of the cylinder, in cubic units? 4πx2 2πx3 πx2 2x 2 πx3

Answers

The expression that represents the volume of the cylinder, in cubic units, is:

[tex]$$V = 2\pi x^3$$[/tex]

The expression that represents the volume of a cylinder in cubic units is given by the formula:

[tex]$$V = \pi r^2h$$[/tex]

where [tex]$r$[/tex] is the radius of the base of the cylinder and [tex]$h$[/tex] is the height of the cylinder.

Now, let's consider each option provided:

[tex]1. $4\pi x^2$[/tex]

This expression only includes the radius, but it does not include the height of the cylinder, so it cannot be the correct answer.

[tex]2. $2\pi x^3$[/tex]

This expression includes both the radius and the height of the cylinder, but it does not include the squared term for the radius, so it cannot be the correct answer.

[tex]3. $\pi x^2$[/tex]

This expression includes the squared term for the radius, but it does not include the height of the cylinder, so it cannot be the correct answer.

[tex]4. $2x$[/tex]

This expression only includes a single variable, which is neither the radius nor the height of the cylinder, so it cannot be the correct answer.

[tex]5. $2\pi x^3$[/tex]

This expression includes both the squared term for the radius and the height of the cylinder, so it is the correct answer.

Therefore, the expression that represents the volume of the cylinder, in cubic units, is:

[tex]$$V = 2\pi x^3$$[/tex]

This formula can be used to calculate the volume of a cylinder given the value of its radius and height.

To learn more about  volume of the cylinder refer here:

https://brainly.com/question/29762858

#SPJ11

The function R = 73. 3*/M3, known as Kielber's law, relates the basal metabolic rate R In Calories per day


burned and the body mass M of a mammal In kilograms.


a. Find the basal metabolic rate for a 180 kilogram lion. Then find the formula's prediction for a 80


kilogram human. If necessary round down to the nearest 50 Calories.


b. Use your metabolic rate result for the lion to find what the basal metabolic rate for a 80 kllogram


human would be if metabolic rate and mass were directly proportional. Compare the result to the result


from part a.


a. Kleiber's law for lion


Calories


Kleiber's law for humans


Calories


b. If metabolic rate and mass were directly proportional


Calories


If the metabolic rate were directly proportional to mass, then the rate for a human would be


(select)


than the actual prediction from Kleiber's law. Kleiber's law Indicates that smaller


organisms have a (select) v metabolic rate per kilogram of mass than do larger organisms.

Answers

The basal metabolic rate for a 180-kilogram lion is approximately 766.4 Calories per day.

The formula predicts that an 80-kilogram human would have a basal metabolic rate of approximately 1,313.9 Calories per day.

The basal metabolic rate is the amount of energy that an organism needs to carry out its basic physiological functions, such as breathing and circulating blood. In this case, Kielber's law is expressed as:

R = 73 [tex]\sqrt[4]{M^3}[/tex]

Let's use this function to find the basal metabolic rate for a 180-kilogram lion. To do this, we simply substitute M = 180 into the equation and solve for R:

R = 73 [tex]\sqrt[4]{180^3}[/tex]

R = 73 [tex]\sqrt[4]{5832}[/tex]

R ≈ 766.4

Now, let's find the formula's prediction for an 80-kilogram human. Again, we simply substitute M = 80 into the equation and solve for R:

R = 73[tex]\sqrt[4]{80^3}[/tex]

R = 73[tex]\sqrt[4]{512}[/tex]

R ≈ 1,313.9

To know more about function here

https://brainly.com/question/28193995

#SPJ4

Complete Question:

The function R = 73 [tex]\sqrt[4]{M^3}[/tex], known as Kielber's law, relates the basal metabolic rate R In Calories per day burned and the body mass M of a mammal In kilograms.

Find the basal metabolic rate for a 180-kilogram lion. Then find the formula's prediction for an 80-kilogram human. If necessary round down to the nearest 50 Calories.

50 PONTS Triangle LMN has vertices at L(−1, 4), M(−1, 0), and N(−3, 4) Determine the vertices of image L′M′N′ if the preimage is rotated 90° clockwise about the origin.

L′(4, 1), M′(0, 1), N′(4, 3)
L′(−1, −4), M′(−1, 0), N′(−3, −4)
L′(−4, −1), M′(0, −1), N′(−4, −3)
L′(1, −4), M′(1, 0), N′(3, −4)

Answers

The coordinates of the resulting triangle are L'(4, 1), M'(0, 1), and N'(4, 3)

What are the coordinates of the resulting triangle?

From the question, we have the following parameters that can be used in our computation:

Triangle LMN has vertices at L(−1, 4), M(−1, 0), and N(−3, 4

This means that

L(−1, 4), M(−1, 0), and N(−3, 4Rotation rule = 90° clockwise around the origin.

The rotation rule of 90° clockwise around the origin is

(x,y) becomes (y,-x)

So, we have

Image = (y, -x)

Substitute the known values in the above equation, so, we have the following representation

L'(4, 1), M'(0, 1), and N'(4, 3)

Hence, the coordinates of the resulting points, are L'(4, 1), M'(0, 1), and N'(4, 3)

Read more about transformation at

brainly.com/question/27224272

#SPJ1

Answer:L′(4, 1), M′(0, 1), N′(4, 3)

Step-by-step explanation:

I am in the middle of taking the quiz and I believe this is the correct answer!

x Which statement about prime and composite numbers is true?
x
A The product of any two prime numbers is a prime number.
* B The product of any two prime numbers is a composite number.
* C All prime numbers are odd numbers.
√x
D All even numbers are composite numbers.

Answers

B, as 2 primes mean another solution to get a composite number. Ex: 7 and 3. Together they make 21, and 21 can be divided into 2 things: 1 and 21 along with 7 and 3

Evaluate the repeated integral: lolla (-xy + 2 z) dz dy dx a) O 15 b) 60 c) 30 d) 36 e) O72 f) O None of these.

Answers

The evaluation of the repeated integral is None of these. (option f)

The repeated integral given is ∫∫∫(-xy + 2z) dz dy dx over the region lolla. This means that you need to integrate the function (-xy + 2z) with respect to z, then with respect to y, and finally with respect to x over the region lolla.

To evaluate this integral, you can use the method of iterated integrals. First, integrate (-xy + 2z) with respect to z, treating x and y as constants:

∫∫(-xy + 2z) dz = -xyz + z² + C

where C is the constant of integration.

Next, integrate the result of the first integral with respect to y, treating x as a constant:

∫[-xyz + z² + C] dy = -xyz + y[-xyz + z² + C] + D

where D is the constant of integration.

Finally, integrate the result of the second integral with respect to x:

∫[-xyz + y(-xyz + z² + C) + D] dx = (-1/2) x² yz + xy(-xyz + z² + C) + Dx + E

where E is the constant of integration.

Now, you need to evaluate this expression over the region lolla. Without further information about the limits of integration for each variable, it is not possible to determine the exact value of this integral.

Therefore, the correct answer is f) None of these.

To know more about integral here

https://brainly.com/question/18125359

#SPJ4

Penelope invested $89,000 in an account paying an interest rate of 6}% compounded continuously. Samir invested $89,000 in an account paying an interest rate of 6⅜% compounded monthly. To the nearest hundredth of a year, how much longer would it take for Samir's money to double than for Penelupe's money to double?

Answers

To solve the problem, we need to find out how much longer it would take for Samir's money to double compared to Penelope's money, given that Penelope invested $89,000 in an account with a continuous interest rate of 6%, while Samir invested $89,000 in an account with a monthly compounded interest rate of 6⅜%.

For Penelope's investment, we can use the formula for continuous compounding, which is A = Pe^(rt), where A is the amount of money after t years, P is the initial investment, r is the interest rate as a decimal, and e is the natural logarithm base. We know that Penelope invested $89,000 and we want to find t such that A = 2P = $178,000. Thus, we have:

$178,000 = $89,000e^(0.06t)

Dividing both sides by $89,000 and taking the natural logarithm of both sides, we get:

ln(2) = 0.06t

Solving for t, we get:

t = ln(2)/0.06 ≈ 11.55 years

For Samir's investment, we can use the formula for monthly compounded interest, which is A = P(1 + r/12)^(12t), where A, P, r are the same as before, and t is the time in years divided by 12. Similarly, we know that Samir invested $89,000 and we want to find t such that A = 2P = $178,000. Thus, we have:

$178,000 = $89,000(1 + 0.0638/12)^(12t)

Dividing both sides by $89,000 and taking the logarithm (base 1 + r/12) of both sides, we get:

log(2)/log(1 + 0.0638/12) = 12t

Solving for t, we get:

t ≈ 11.80/12 = 0.98 years

To find the difference in time it takes for Samir's money to double compared to Penelope's, we subtract the time it takes for Penelope's money to double from the time it takes for Samir's money to double:

0.98 - 11.55 ≈ -10.57

However, this answer doesn't make sense in the context of the problem, since it's negative. After reviewing our solution, we realized that we made a mistake in the calculation of t for Penelope's investment. We need to find the time it takes for Penelope's investment to double with annual compounding, not continuous compounding. The formula for this is t = (ln(2))/(ln(1 + r)), where r is the annual interest rate as a decimal.

Plugging in the numbers, we get:

t = (ln(2))/(ln(1 + 0.06)) ≈ 11.55 years

This is the same as the time we got for Samir's investment, so the difference in time it takes for their money to double is:

0.98 - 11.55 ≈ -10.57

Again, this answer doesn't make sense in the context of the problem, since it's negative. Therefore, we need to revise our solution and approach the problem differently.

Select all the expressions that are equivalent to –25 (fraction 2 over 5)
(15 – 20d).

Answers

The equivalent expressions are,

A. [tex]-30 + 40d - 10c[/tex],

B. [tex]-6 + 8d - 2c[/tex]

D. [tex]6 - 8d + 2c[/tex].

What are expressions?

An expression is a sentence with at least two numbers or variables having mathematical operation. Math operations can be addition, subtraction, multiplication, division.

For example, [tex]2x+3[/tex]

The given expression.

[tex]\implies -25(15 - 20d + 5c)[/tex]

[tex]\implies -125(3 - 4d + c)[/tex]

So, the given expression can be converted into

[tex]k(3 - 4d + c)[/tex]

The equivalent expressions are:

A. [tex]-30 + 40d - 10c[/tex],

B. [tex]-6 + 8d - 2c[/tex]

D. [tex]6 - 8d + 2c[/tex].

A. [tex]-30 + 40d - 10c[/tex]

[tex]\implies -30 + 40d - 10c[/tex]

[tex]\implies -10(3 - 4d + c)[/tex]

B. [tex]-6 + 8d - 2c[/tex]

[tex]\implies -6 + 8d - 2c[/tex]

[tex]\implies -2(3 - 4d + c)[/tex]

D. [tex]6 - 8d + 2c[/tex]

[tex]\implies6 - 8d + 2c[/tex]

[tex]\implies2(3 - 4d + c)[/tex]

To know more about Expressions check:

brainly.com/question/16804733

Lines ab and cd are parallel. if 6 measures (4x - 31)°, and 5 measures 95°, what is the value of x? a. x = 19 b. x = 95 c. x = 265 d. x = 29

Answers

Answer: x=29

Step-by-step explanation:

To find the value of x, we can set the two angles equal to each other and solve for x, which gives x = 19.

What will be the value of x if 6 measures (4x - 31)° and 5 measures 95° in parallel lines ab and cd?

We can use the fact that alternate interior angles are congruent when a transversal intersects parallel lines. In this case, line ab and cd are parallel and 6 and 5 are alternate interior angles. So we can set up an equation:

4x - 31 = 95

Solving for x:

4x = 126

x = 31.5

So the value of x is not one of the answer choices given. However, if we round x to the nearest integer, we get x = 32, which is closest to answer choice (d) x = 29. Therefore, the closest answer choice is (d) x = 29.

Learn more about interior angles

brainly.com/question/10638383

#SPJ11

How do I take a picture

Answers

To take a picture we must press the shutter button pointing the lens towards the image we want to capture.

How to take a picture?

To take a picture we must follow the following steps. In general, we must have a camera at hand and know how to use it. There is a great diversity of cameras with different characteristics, but the basics to take a photo are the following:

In the first place, we must locate ourselves at a prudent distance from the element that we are going to photograph, making sure that it comes out completely in the camera's focus.

Once we have focused on the object, we must make sure that nothing is going to move the camera or go through between the camera and the object.

Later, we must make sure that there is enough light for the object to come out sharp in the photo.

Finally, we press the shutter and take the photo. In some cases we will have the digital photo or in others we will be able to print it on photographic paper.

Learn more about photography at: https://brainly.com/question/30685203

#SPJ1

Find the midpoint of the line segment joining (-4,-2) and (2,8) please show how u got your answer!!!

Answers

Answer:

(1,3)

Step-by-step explanation:

The midpoint formula is:

[tex](\frac{x1+x2}{2}),(\frac{y1+y2}{2})[/tex]

We have our 2 points, (-4,-2) and (2,8).

For this sake and for this explanation, point 1 is (-4,-2), and point 2 is (2,8).

We can substitute in our values:

[tex](\frac{-4+2}{2}),(\frac{-2+8}{2})[/tex]

substitute

[tex](\frac{2}{2}),(\frac{6}{2})[/tex]

Our midpoint is located at (1,3)

Hope this helps! :)

Answer:

(-1,3)

Step-by-step explanation:

To find the midpoint of a line segment, you want to find the change in x and y.

From (-4,-2) to (2,8), you move right by 6 and up by 10.

The midpoint is exactly half of this, meaning right by 3 and up by 5.

Therefore, the midpoint is (-1,3).

1. Sally Rose's charge account statement showed a previous balance of $6,472. 82, a finance charge of $12. 95,


new purchases of $1,697. 08, and a payment of $4,900. 50. What is her new balance?


a. $3,454. 99


c. $3,566. 44


b. $3,282. 35


d. $3,112. 78

Answers

Sally Rose's new balance is $3,282.35, and option (b) is the correct answer.

Sally Rose's charge account statement contains information about her previous balance, finance charge, new purchases, and payment. To determine her new balance, we need to take the previous balance, add the finance charge and new purchases, and then subtract the payment.

Starting with the previous balance of $6,472.82, we add the finance charge of $12.95 and new purchases of $1,697.08 to get a total of:

$6,472.82 + $12.95 + $1,697.08 = $8,182.85

Next, we subtract the payment of $4,900.50 to get the new balance:

$8,182.85 - $4,900.50 = $3,282.35

It's important to keep track of credit card balances to avoid accumulating too much debt and paying high interest charges. When making credit card payments, it's a good idea to pay more than the minimum amount due, which can help reduce the balance faster and save money on interest charges over time. The answer is option b).

Know more about new balance here:

https://brainly.com/question/8549088

#SPJ11

A jury of 6 persons was selected from a group of 20 potential jurors, of whom 8 were african american and 12 were white. the jury was supposedly randomly selected, but it contained only 1 african american member. a) do you have any reason to doubt the randomness of the selection

Answers

Yes, there is reason to doubt the randomness of the jury selection based on the information provided.

Given data:

Out of the 20 potential jurors, 8 were African American and 12 were white. The probability of randomly selecting an African American juror from the pool of potential jurors would ideally be 8/20, which simplifies to 2/5 or 40%. However, the actual jury selected had only 1 African American member out of 6 jurors, which is significantly lower than the expected 40% if the selection were truly random.

This deviation from the expected probability raises questions about the randomness of the selection process. The observed outcome appears to be disproportionately skewed against the representation of African American jurors. While random variations can occur, the extent of the deviation in this case warrants further investigation into the jury selection process to determine if there were any biases or factors influencing the outcome.

To learn more about probability, refer:

https://brainly.com/question/17089724

#SPJ12

At the toy store, 4 toy cars cost $3.24. How much does it cost to buy 25 toy cars?=

Answers

Answer:

Each car cost $0.81, you would need to do 0.81 times 25 and you would get $20.25

Step-by-step explanation:

Qn in attachment
.
..​

Answers

Answer:

option a

Step-by-step explanation:

it is the formula for varience.

Other Questions
What is the preposition for "ivan borrowed the books from me" Describe how a manager who derives satisfaction from both income and shirking allocates a 10-hour day between these activities when paid an annual, fixed salary of $135,000. Time spent working: hours Time spent shirking: hours When this same manager is given an annual, fixed salary of $135,000 and 4 percent of the firms profitsamounting to a total salary of $155,000 per yearthe manager chooses to work 8 hours and shirks for 2 hours. Given this information, which of the compensation schemes does the manager prefer? multiple choice The scheme with fixed payment of $135,000 and a percentage of profits. The scheme with only a fixed payment of $135,000. The manager is indifferent between the two payment schemes How many lead atoms are present in a piece of lead of volume 1. 907 cm?Useful data:The density of Pb = 11. 34 g. Cm-3. Avogadro's Number (NA) = 6. 022x1023 atoms. Mol-1Give your answer to 3 significant figuresGiving your answer with specific significant figures. Use 'e' notation for powers of ten:for example for 1. 23 x 104, enter 1. 23e4for example for 6. 022 x 1023, enter 6. 022e23for example for 1. 23 x 10-6, that is 0. 00000123, enter 1. 23e-6note no gaps 8 unit 8 quiz SOWhich statement BEST explains why sociologists are particularly interested in social changes that came with industrialization? A. They were often the ones who predicted social change. B. There are fewer industrial cities, so it is easier to study them. C. Early sociologists could not access rural communities as easily. D. Societies that have not industrialized tend to change more slowly. The percent of US citizens that use the internet on a daily basis is shown on the graph, where the year 2000 corresponds to x=0. Use the concepts of y-intercept and slope of the line to find the equation that best models the given data. Answer in slope intercept form. Why is the krebs cycle so important if it only produces 2 atp molecules. The holiday health club has reduced its annual membership by 10%. if jorge sanchez purchases a years membership and pays $270, what is the regular membership fee? If you had 3. 8 x 10^22 J of energy and you had a machine that could turn all of that energy into mass, what would be your mass in kg? o produto de dois nmeros 54 seu MMC 18 Qual o MDC desse nmero?Porfavor explique A machine has an initial cost of $40,000 and operating costs of $3500 each year. Its salvage value decreases by $4,000 each year. The machine is now 4 years old.Assuming an effective annual interest rate of 12%, what is the cost of owning and operating the machine for one more year. 50 POINTS!! HELP I dont understand how in this world the answers came to this so i was hoping if someone could explain it me pls 50 points :) look at picture below can i claim my child as a dependent if they file a tax return 35BSBHRM614_Learner Workbo...TIONAReport Template (completed)You will play the role of Zane O'Brien to research and analyse futureworkforceneeds at CBSA.Based on the email, review the staff audit results on staff turnover anddemographics, the summary of the Strategic Plan which contains the strategicobjectives of the organisation, and the organisation's policies and proceduresincluding the BD001 Business Plan. Then using the Report Template, developandsubmit a report where you are to:1. Detail workforce trend analysis of the organisation. You should provide:an overview of the departments that CBSA usesan overview of the current number of employees including theiremploymenttype (full-time, part-time, etc.)the services CBSA currently offers and those that it intends to offer(. analysis of the demographics and education level of the employeesanalysis of staff turnover from the last yearthe number of new human resources required for the coming year.2. Detail the future workforce needs by outlining: the job rolethe number of employees required for the job role the skills required for the job rolewhat option(s) should be used to source the job role.3. Detail the technology considerations by outlining:the current technology used by the organisationany different technology that could be implemented to increaseproductivitywhich may impact on job roles/design/skills.4. Detail legislation and industrial considerations by outlining:an overview of the major policies and procedures used by theorganisationand the legislation they relate toresearch and detail any recent or impending legislation orindustrialchanges that may affect the organisation in somecapacity. 5) The I in C+I+G+(X-M) in gdp represents which of the following? Luther Burbank bred potatoes that combined a variety of useful traits. One of Burbank's breeding techniques was hybridization. Which cross ofpotato plants would have been MOST USEFUL for producing a useful strain by hybridization?O A a disease-resistant plant crossed with a plant that is not disease resistantO B. A plant that is not disease resistant crossed with a plant that produces a low volume of foodO c. A disease-resistant plant crossed with a plant that produced a low volume of foodO D. A disease-resistant plant crossed with a plant that produced a high volume of food What are 3 goals you have to improve your technology skill throughout out work 100 Solve 2^{3-x} + 2^{x+1} =17Log x + log (x+3) = 1 E8-7 (algo) computing bad debt expense using aging of accounts receivable method [lo 8-2] brown cow dairy uses the aging approach to estimate bad debt expense. the balance of each account receivable is aged on the basis of three time periods as follows: (1) 1 to 30 days old, $12,900; (2) 31 to 90 days old, $5,900; and (3) more than 90 days old, $3,900. for each age group, the average loss rate on the amount of the receivable due to uncollectibility is estimated to be (1) 5 percent, (2) 10 percent, and (3) 15 percent, respectively. at december 31 (end of the current year), the allowance for doubtful accounts balance was $890 (credit) before the end-of-period adjusting entry is made. required: prepare a schedule to estimate an appropriate year-end balance for the allowance for doubtful accounts. what amount of bad debt expense should be recorded on december 31 How many grams of table salt are made from the synthesis reaction of chlorine gas and 400 grams of sodium metal? A 5. 00-g sample of aluminum pellets (Cs = 0. 89 J/gC) and a 10. 00-g sample of iron pellets (Cs= 0. 45 J/gC) are heated to 100. 0C. The mixture of hot iron and aluminum is then dropped into an unknown mass of water (Cs= 4. 18 J/gC) at 22. 0C. The final temperature of the water and metals mixture is 23. 7C. How much heat (in J) is transferred to the water by aluminum pellets? I am confused how to determine the mass of water